Line 5: Line 5:
 
This is clearly a measure on <math>A</math> with <math>\lambda(A)=1 \frac{}{}</math>
 
This is clearly a measure on <math>A</math> with <math>\lambda(A)=1 \frac{}{}</math>
  
Moreover, <math>\int_{A}fd\mu = \mu(A)\int_A f d\lambda \frac{}{}</math>
+
Moreover, <math>\int_{A}fd\mu = \mu(A)\int_A f d\lambda \frac{}{}</math>.
 +
 
 +
By Jensen's Inequality, we get
 +
 
 +
<math>\phi(\int_Afd\lambda \leq \int_A\phi(f)d\lambda</math>

Revision as of 10:49, 22 July 2008

Define a function from the set of all measurable subset $ B $ of $ A $ as below

$ \lambda(B)=\frac{\mu(B)}{\mu(A)} $

This is clearly a measure on $ A $ with $ \lambda(A)=1 \frac{}{} $

Moreover, $ \int_{A}fd\mu = \mu(A)\int_A f d\lambda \frac{}{} $.

By Jensen's Inequality, we get

$ \phi(\int_Afd\lambda \leq \int_A\phi(f)d\lambda $

Alumni Liaison

To all math majors: "Mathematics is a wonderfully rich subject."

Dr. Paul Garrett